LSAT and Law School Admissions Forum

Get expert LSAT preparation and law school admissions advice from PowerScore Test Preparation.

 Adam Tyson
PowerScore Staff
  • PowerScore Staff
  • Posts: 5153
  • Joined: Apr 14, 2011
|
#67310
This is the second question about this stimulus, the first being #17, a Point at Issue question. This second question is a Method of Reasoning question, which is about describing the strategy used by Smith to respond to Jones. Your prephrase should describe what Smith did, rather than what Smith said.

So, what did Smith do? He pointed out Jones overlooked something in his analysis. What did Jones overlook? The possibility that there could have been migrating people using wooden tools of the same type found in South America that left no trace of such along the route. Smith points out that the tools were found in peat bogs, which preserved them, and told us that peat bogs are rare in the Americas.

Put another way, Jones assumed that if those sorts of tools had been used by those migrating ancestors, we would have found some evidence of them somewhere between Alaska and South America. Smith points out that this assumption could be incorrect.

That is what answer E is describing for us - an "implicit assumption" is, well, an assumption! Smith challenged Jones' assumption that we would have found evidence of those tools along the route if the scientists were correct.

Answer A is incorrect because Smith cited no studies.

Answer B is incorrect because Smith did not accuse Jones of doing anything wrong, other than fail to consider the important role of peat bogs in preserving the tools in South America.

Answer C is incorrect because Smith does not "disrupt the accuracy" (that's an interesting turn of phrase!) of Jones' evidence. He is fine with Jones' evidence - it's apparently true that we have not found any evidence of those tools along the migration route. The evidence is accurate! The problem is that the evidence doesn't prove what Jones thinks it proves, because he bases his conclusion on a faulty premise.

Answer D is incorrect because Jones' evidence doesn't support a contrary conclusion. Again, the evidence is fine! It helps Jones, for certain. It just isn't sufficient to make Jones' case for him.
 andriana.caban
  • Posts: 142
  • Joined: Jun 23, 2017
|
#67317
I checked out the Point At Issue question. Interesting that a Stimulus could have two questions about it (I haven't started doing PT's yet so maybe I'll see more questions like that).

Thanks for your explanation - it makes sense now.

Get the most out of your LSAT Prep Plus subscription.

Analyze and track your performance with our Testing and Analytics Package.